Difference between revisions of "User:Nik"

From Ferienserie MMP2
Jump to: navigation, search
m (Problem)
 
(60 intermediate revisions by 4 users not shown)
Line 1: Line 1:
== Foreword ==
+
Niklaus Messerli
I use \(Q\:/\:P\:/\:H\) for the transformed system instead of \(\widetilde{q}\:/\:\widetilde{p}\:/\:\widetilde{h}\) because it's easier to write in Latex.  
+
nik@student.ethz.ch
== Problem ==
+
Let
+
\( \Phi \in C^\infty(\mathbb{R}^n) \)
+
have the property that the system
+
\( p_i = \frac{\partial}{\partial q_i} \Phi (q, Q) \)
+
has a unique smooth solution
+
\( Q = Q(q,p) \).  
+
  
Define
+
{| class="wikitable"
\( P_i(q,p) = - \frac{\partial}{\partial Q_i} \Phi (q, Q) | _{Q= Q(q,p)} \)
+
!Article
 
+
!Check
Let \( \{\cdot,\cdot\} \) be the Poisson bracket, such that
+
|-
\(  \{f,g\} = \sum_{j=1}^n \frac{\partial f}{\partial q_j} \frac{\partial g}{\partial p_j} - \frac{\partial f}{\partial p_j} \frac{\partial g}{\partial q_j} \)
+
| [[Aufgaben:Problem 1|Problem 1]]
 
+
| 1
Show that:
+
|-
 
+
| [[Aufgaben:Problem 2|Problem 2]]
\( \{Q_i(q,p), Q_j(q,p)\} = \{P_i (q,p), P_j(q,p)\} = 0 \)
+
| a-c):1, d):0
\( \{Q_i(q,p), P_j(q,p)\} = \delta_{ij} \)
+
|-
 
+
| [[Aufgaben:Problem 3|Problem 3]]
==Solution==
+
| d) Wieso müssen wir zeigen dass "\(\{L_g: g \in G\}\) is a group under composition"? Wir wissen doch schon, dass \(L: G \rightarrow \mathrm{Sym}G\) und dass \(G\) und \(\mathrm{Sym}G\) Gruppen sind.
===Assumptions===
+
|-
From the script we use the hamilton's equations \(\dot{q_i} = \{q_i, h\} = \frac{\partial h}{\partial p_i} \text{ and } \dot{p_i} = \{p_i, h\} = -\frac{\partial h}{\partial q_i} \)
+
| [[Aufgaben:Problem 4|Problem 4]]
 
+
| 1
 
+
|-
Therefore (to show) $$ \tag{1} \dot{Q_i} = \{Q_i, h\} = \frac{\partial H}{\partial P_i} $$
+
| [[Aufgaben:Problem 5|Problem 5]]
and $$ \tag{2} \dot{P_i} = \{P_i, h\} = -\frac{\partial H}{\partial Q_i} $$
+
| 1
where \( H = h\circ\Phi \)
+
|-
 
+
| [[Aufgaben:Problem 6|Problem 6]]
The question if we are allowed to use this is still not clear. Waiting for an answer of Raisa (02.01.15 - 10:00)
+
| 1
 
+
|-
 
+
| [[Aufgaben:Problem 7|Problem 7]]
 
+
| 1
We also use that $$ \tag{3}  \frac{\partial h}{\partial p_l} = \sum_{k=1}^n(\frac{\partial H}{\partial Q_k}\frac{\partial Q_k}{\partial p_l} + \frac{\partial H}{\partial P_k}\frac{\partial P_k}{\partial p_l}) $$
+
|-
and $$ \tag{4}  \frac{\partial h}{\partial q_l} = \sum_{k=1}^n(\frac{\partial H}{\partial Q_k}\frac{\partial Q_k}{\partial q_l} + \frac{\partial H}{\partial P_k}\frac{\partial P_k}{\partial p_l}) $$
+
| [[Aufgaben:Problem 8|Problem 8]]
which is just using the chain rule.
+
| 2
 
+
|-
===Shaking variables===
+
| [[Aufgaben:Problem 9|Problem 9]]
With these definitions it's just about shaking and changing sums:
+
| 1
 
+
|-
$$ \dot{Q_i} = \{Q_i, h\} = \sum_{l=1}^n(\frac{\partial Q_i}{\partial q_l}\frac{\partial h}{\partial p_l} - \frac{\partial Q_i}{\partial p_l}\frac{\partial h}{\partial q_l}) $$
+
| [[Aufgaben:Problem 10|Problem 10]]
With (3) and (4) it follows
+
| a by craven seems legit
$$ = \sum_{l=1}^n(\frac{\partial Q_i}{\partial q_l}\sum_{j=1}^n(\frac{\partial H}{\partial Q_j}\frac{\partial Q_j}{\partial p_l} + \frac{\partial H}{\partial P_j}\frac{\partial P_j}{\partial p_l}) - \frac{\partial Q_i}{\partial p_l}\sum_{j=1}^n(\frac{\partial H}{\partial Q_j}\frac{\partial Q_j}{\partial q_l} + \frac{\partial H}{\partial P_j}\frac{\partial P_j}{\partial p_l})) $$
+
|-
Now we exchange the sums
+
| [[Aufgaben:Problem 11|Problem 11]]
$$ = \sum_{j=1}^n(\frac{\partial H}{\partial Q_j}\sum_{l=1}^n(\frac{\partial Q_i}{\partial q_l}\frac{\partial Q_j}{\partial p_l} - \frac{\partial Q_i}{\partial p_l}\frac{\partial Q_j}{\partial q_l}) + \frac{\partial H}{\partial Q_j}\sum_{l=1}^n(\frac{\partial Q_i}{\partial q_l}\frac{\partial P_j}{\partial p_l} - \frac{\partial Q_i}{\partial p_l}\frac{\partial P_j}{\partial q_l})) $$
+
| 1
And with the definition of the Poisson-brackets
+
|-
$$ = \sum_{j=1}^n(\frac{\partial H}{\partial Q_j}\{Q_i, Q_j\} + \frac{\partial H}{\partial Q_j}\{Q_i, P_j\}) $$
+
| [[Aufgaben:Problem 12|Problem 12]]
To fulfill (1)
+
| bitch please
$$ \implies \{Q_i, Q_j\} = 0 \text{ and } \{Q_i, P_j\} = \delta_{ij} $$
+
|-
 
+
| [[Aufgaben:Problem 13|Problem 13]]
And the same for the other equation:
+
|
 
+
|-
$$ \dot{P_i} = \{Q_i, h\} = \sum_{l=1}^n(\frac{\partial P_i}{\partial q_l}\frac{\partial h}{\partial p_l} - \frac{\partial P_i}{\partial p_l}\frac{\partial h}{\partial q_l}) $$
+
| [[Aufgaben:Problem 14|Problem 14]]
With (3) and (4) it follows
+
|
$$ = \sum_{l=1}^n(\frac{\partial P_i}{\partial q_l}\sum_{j=1}^n(\frac{\partial H}{\partial Q_j}\frac{\partial Q_j}{\partial p_l} + \frac{\partial H}{\partial P_j}\frac{\partial P_j}{\partial p_l}) - \frac{\partial P_i}{\partial p_l}\sum_{j=1}^n(\frac{\partial H}{\partial Q_j}\frac{\partial Q_j}{\partial q_l} + \frac{\partial H}{\partial P_j}\frac{\partial P_j}{\partial p_l})) $$
+
|-
Now we exchange the sums
+
| [[Aufgaben:Problem 15|Problem 15]]
$$ = \sum_{j=1}^n(\frac{\partial H}{\partial Q_j}\sum_{l=1}^n(\frac{\partial P_i}{\partial q_l}\frac{\partial Q_j}{\partial p_l} - \frac{\partial P_i}{\partial p_l}\frac{\partial Q_j}{\partial q_l}) + \frac{\partial H}{\partial P_j}\sum_{l=1}^n(\frac{\partial P_i}{\partial q_l}\frac{\partial P_j}{\partial p_l} - \frac{\partial P_i}{\partial p_l}\frac{\partial P_j}{\partial q_l})) $$
+
| a): 1
And with the definition of the Poisson-brackets
+
|-
$$ = \sum_{j=1}^n(\frac{\partial H}{\partial Q_j}\{P_i, P_j\} + \frac{\partial H}{\partial P_j}\{P_i, P_j\}) $$
+
|}
To fulfill (2)
+
$$ \implies \{P_i, P_j\} = 0 \text{ and } \{P_i, Q_j\} = -\delta_{ij} =  -\{Q_i, P_j\} \:\:\:\blacksquare $$
+
 
+
==Attempts==
+
- Use of hamilton's equations from script page 59. I asked Raisa if we are allowed to use things from the script without proof, no answer yet (31.12.14 - 21:00)
+
The problem there was, that you have to use a hamiltonian from the new coordinate system. May work, haven't checked that exactly.
+
 
+
- Use of simply the chain rule and insert all the things. Some solutions i saw with that said that \( \frac{\partial \Phi}{\partial p} = 0 \) but since \(\Phi \) depends on \(Q \) whicht depends on \(p \) I am not convinced with that.
+
 
+
- Use of simply the chain rule as before. At some point of this solution you should show that \( \frac{\partial Q_a}{\partial q_j}\frac{\partial}{\partial p_j}\frac{\partial}{\partial Q_j}\Phi (q, Q) =  0 \) which nobody has until now.
+

Latest revision as of 14:13, 3 August 2015

Niklaus Messerli nik@student.ethz.ch

Article Check
Problem 1 1
Problem 2 a-c):1, d):0
Problem 3 d) Wieso müssen wir zeigen dass "\(\{L_g: g \in G\}\) is a group under composition"? Wir wissen doch schon, dass \(L: G \rightarrow \mathrm{Sym}G\) und dass \(G\) und \(\mathrm{Sym}G\) Gruppen sind.
Problem 4 1
Problem 5 1
Problem 6 1
Problem 7 1
Problem 8 2
Problem 9 1
Problem 10 a by craven seems legit
Problem 11 1
Problem 12 bitch please
Problem 13
Problem 14
Problem 15 a): 1